Đến nội dung

thanhdotk14 nội dung

Có 265 mục bởi thanhdotk14 (Tìm giới hạn từ 27-04-2020)



Sắp theo                Sắp xếp  

#567997 Có 20 hoc sinh được chia thành 10 tổ, mỗi tổ 2 học sinh. Hỏi có bao nhiêu các...

Đã gửi bởi thanhdotk14 on 24-06-2015 - 23:16 trong Tổ hợp - Xác suất và thống kê - Số phức

Đề bài  Có 20 hoc sinh được chia thành 10 tổ, mỗi tổ 2 học sinh. Hỏi có bao nhiêu cách chia .
---------

Tâm sự đêm khuya cùng Kiến sida =))




#557296 chứng minh bất đẳng thức holder dạng$(a^{3}+b^{3}+c^...

Đã gửi bởi thanhdotk14 on 01-05-2015 - 17:12 trong Bất đẳng thức và cực trị

Bất đẳng thức Holder có rất nhiều tài liệu viết mà, các em có thể tìm kiếm nhiều cách chứng minh khá hay trên mạng :)




#526389 Đề thi chọn HSG trường THPT chuyên Lê Quý Đôn, Bình Định 2014-2014

Đã gửi bởi thanhdotk14 on 27-09-2014 - 17:41 trong Thi HSG cấp Tỉnh, Thành phố. Olympic 30-4. Đề thi và kiểm tra đội tuyển các cấp.

Lười gõ quá, nên up ảnh thôi nhá :D :D :D

10580480_369526263204390_889474448_n.jpg

_____

@Huy: rồi đó nhé :3




#487608 Đề thi học sinh giỏi lớp 11 THPT tỉnh Bình Định năm học 2014

Đã gửi bởi thanhdotk14 on 18-03-2014 - 18:48 trong Thi HSG cấp Tỉnh, Thành phố. Olympic 30-4. Đề thi và kiểm tra đội tuyển các cấp.

KỲ THI CHỌN HỌC SINH GIỎI CẤP TỈNH LƠP 11 THPT 

NĂM HỌC: 2013-2014

______________________

 

 

 

 

Bài 1:

      $1.$ Giải phương trình: $$x^3+\sqrt{(1-x^2)^3}=x\sqrt{2-2x^2}$$

      $2.$ Cho $x,y,z>0$ và $xyz=1$. 

              Tìm giá trị nhỏ nhất của biểu thức: $$M=\frac{1}{x^3(y+z)}+\frac{1}{y^3(z+x)}+\frac{1}{z^3(x+y)}$$

Bài 2:

      $1.$ Chứng minh rằng với $n$ là số tự nhiên chẵn thì tổng $T$ sau chia hết cho $2^n$

$$T=C_{2n}^0+5C_{2n}^2+5^2C_{2n}^4+...+5^{i}C_{2n}^{2i}+...+5^{n}C_{2n}^{2n}$$

      $2.$ Cho dãy số xác định bởi:

$$\left\{\begin{matrix} u_1=-1 & & \\ u_n=\frac{u_{n-1}+\sqrt{3}}{1-\sqrt{3}.u_{n-1}} ,n=2,3,...& & \end{matrix}\right.$$

          $a.$ Lập công thức tổng quát của dãy số $(u_n)$.

          $b.$ Tính: $S_{2014}=u_1+u_2+...+u_{2014}$

Bài 3:

        Cho bốn số thực $a,b,c,d$ thỏa các hệ thức: $a^2+b^2=1$ và $c+d=4$

         Tìm giá trị lớn nhất của $P=ac+bd+cd$

Bài 4:

        Cho tam giác đều $OAB$ cạnh a. Trên đường thẳng $d$ qua $O$ và vuông góc với mặt phẳng $(OAB)$, lấy $M$ sao cho $OM=x$. Gọi $E,F$ là các hình chiếu của A lên $MB$ và $OB$. Gọi $N$ là giao điểm $EF$ và $d$. Xác định $x$ để thể tích tứ diện $ABMN$ nhỏ nhất.

___________

Hi vọng trường mình làm tốt các bài này :)




#485753 Cho $a,b,c>0$ sao cho abc=1. Chứng minh rằng: $\sum...

Đã gửi bởi thanhdotk14 on 04-03-2014 - 06:15 trong Bất đẳng thức - Cực trị

Cho $a,b,c>0$ sao cho $abc=1$. Chứng minh rằng: $\sum \frac{a^3+5}{a^3(b+c)}\ge9$

Ta có: $$\frac{a^3+5}{a^3(b+c)}\ge \frac{3(a+1)}{a^3(b+c)}=\frac{3(1+bc)}{a^2(b+c)}$$

Do đó, ta cần chứng minh:$$\sum \frac{1+bc}{a^2(b+c)}\ge 3$$

$$\Leftrightarrow \sum \frac{1}{a^2(b+c)}+\sum \frac{bc}{a^2(b+c)}\ge 3$$

Lại có: $$\sum \frac{1}{a^2(b+c)}=\sum \frac{bc}{ab+ca}\ge \frac{3}{2}$$

$$\sum \frac{bc}{a^2(b+c)}=\frac{\frac{1}{a^2}}{\frac{1}{b}+\frac{1}{c}}\ge \frac{\frac{1}{a}+\frac{1}{b}+\frac{1}{c}}{2}\ge \frac{3}{2}$$

Từ đó, bất đẳng thức được chứng minh

Đẳng thức xảy ra $\Leftrightarrow a=b=c=1$




#465023 KỲ THI CHỌN ĐỘI TUYỂN HỌC SINH GIỎI VÒNG 2 TỈNH BÌNH ĐỊNH NĂM 2013

Đã gửi bởi thanhdotk14 on 18-11-2013 - 12:33 trong Thi HSG cấp Tỉnh, Thành phố. Olympic 30-4. Đề thi và kiểm tra đội tuyển các cấp.

KỲ THI CHỌN ĐỘI TUYỂN HỌC SINH GIỎI VÒNG 2 TỈNH BÌNH ĐỊNH NĂM 2013

____________________________________

 

 

Bài 1: Giải hệ phương trình: $$\left\{\begin{matrix} x\sqrt{1-y^2}+y\sqrt{1-x^2}=1 & & \\ (1-x)(1+y)=2& & \end{matrix}\right.$$

Bài 2: Chứng minh rằng: Nếu $a,b,c,d$ là bốn nghiệm của đa thức $P(x)=x^4+x^3-1$ thì $ab$ là nghiệm của đa thức $Q(x)=x^6+x^4+x^3-x^2-1$

Bài 3: Cho $a$ và $b$ là hai số nguyên dương.

Chứng minh rằng: Nếu $ab(5a^2+5b^2-2)$ chia hết cho $5ab-1$ thì $a=b$

Bài 4: Cho dãy số $(x_n)$ được xác định như sau:$$x_n=\frac{(2+\sqrt{3})^n-(2-\sqrt{3})^n}{2\sqrt{3}},n\in\mathbb{N^*}$$

$a.$ Chứng minh rằng dãy $(x_n)$ nguyên 

$b.$ Xác định $n$ để $x_n$ chia hết cho $3$

Bài 5:

$a.$ Cho ba điểm $A,B,C$ thẳng hàng ($B$ nằm giữa $A$ và $C$). Gọi $(O)$ là đường tròn đi qua hai điểm $A,C$ ($AC$ không phải là đường kính). Hai tiếp tuyến với $(O)$ tại $A$ và $C$ cắt nhau ở $P$, $PB$ cắt $(O)$ tại $Q$. Phân giác của góc $AQC$ cắt $AC$ tại $R$.

Chứng minh rằng: $\frac{AB}{BC}=\frac{AR^2}{RC^2}$

$b.$ Cho tam giác $ABC$ nội tiếp đường tròn $(O)$ và ngoại tiếp đường tròn $(I)$.Giao điểm của $AI$ với $BC$ và $(O)$ lần lượt là $D,A_1$ ($A_1\ne A$)

Chứng minh rằng: $\frac{IA_1}{DA_1}=\frac{AB+AC}{BC}$




#462706 $ (a+b-c-1)(b+c-a-1)(c+a-b-1) \le 8 $

Đã gửi bởi thanhdotk14 on 07-11-2013 - 18:15 trong Bất đẳng thức - Cực trị

đặt $f(x)=x^3$ => $f'(x)=6x \geq 0$

mà bộ $(4,4,2,0,0)$ trội hơn bộ $(a_1,a_2,a_3,a_4,a_5)$ (hiển nhiên)

nên $f(4)+f(4)+f(2)+f(0)+f(0)\geq f(a_1)+f(a_2)+f(a_3)+f(a_4)+f(a_5)$ (theo bất đẳng thức karamata)

<=> $136 \geq a_1^3+a_2^3+a_3^3+a_4^3+a_5^3$

dấu '=' xảy ra khi a1=a2=4, a3=2,a4=a5=0 và các hoán vị của nó

Đề nghị em chứng minh rõ rành nha  :icon6:  :icon6:  :icon6:

_______________________________________

Bài giải:

Không mất tính tổng quát, ta có thể giả sử:$4\ge a_1\ge a_2\ge a_3\ge a_4\ge a_5\ge 0$

Từ đó suy ra:$$\left\{\begin{matrix} a_1\le 4 & & \\ a_1+a_2\le 8 & & \\ a_1+a_2+a_3\le 10 & & \\ a_1+a_2+a_3+a_4\le 10 & & \\ a_1+a_2+a_3+a_4+a_5=10 & & \end{matrix}\right.$$

Ta có:$$P=a_1^3+a_2^3+a_3^3+a_4^3+a_5^3$$

$$=a_1(a_1^2-a_2^2)+(a_1+a_2)(a_2^2-a_3^2)+(a_1+a_2+a_3)(a_3^2-a_4^2)+(a_1+a_2+a_3+a_4)(a_4^2-a_5^2)+(a_1+a_2+a_3+a_4+a_5)a_5^2$$

$$\le 4(a_1^2-a_2^2)+8(a_2^2-a_3^2)+10(a_3^2-a_4^2)+10(a_4^2-a_5^2)+10a_5^2$$

$$=2(2a_1^2+2a_2^2+a_3^2)$$

Lại có:$$a_1^2+a_2^2+a_3^2=a_1(a_1-a_2)+(a_1+a_2)(a_2-a_3)+(a_1+a_2+a_3)a_3$$

$$\le 4(a_1+a_2)+2a_3=2(a_1+a_2)+2(a_1+a_2+a_3)\le 36$$

Từ đó suy ra:$$P\le 2(36+32)=136$$

Đẳng thức xảy ra $\Leftrightarrow a_1=a_2=4,a_3=2,a_4=a_5=0$ và các hoán vị




#462416 $ (a+b-c-1)(b+c-a-1)(c+a-b-1) \le 8 $

Đã gửi bởi thanhdotk14 on 05-11-2013 - 23:23 trong Bất đẳng thức - Cực trị

Góp vui một bài, bài này khá độc đáo =))

________________________________

Bài 20: Cho các số thực $a_1,a_2,a_3,a_4,a_5\in [0;4]$ và thỏa mãn:$$a_1+a_2+a_3+a_4+a_5=10$$

Tìm $\max$ của biểu thức:$$P=a_1^3+a_2^3+a_3^3+a_4^3+a_5^3$$




#462336 2. Chứng minh: $I\in PQ$

Đã gửi bởi thanhdotk14 on 05-11-2013 - 19:50 trong Hình học

Đường tròn $\left ( E \right )$ tiếp xúc với 2 cạnh $AB, AC$ của $\Delta ABC$ tại $P,Q$ và cũng tiếp xúc trong với đường tròn ngoại tiếp tam giác đó tại $S$. Gọi $(I)$ là đường tròn nội tiếp  $\Delta ABC$.

1. Gọi M trung điểm $BC$. Chứng minh: $\angle BIM=\angle SBI$ .

2. Chứng minh: $I\in PQ$

Đây chính là kết quả của bài toán này nè bạn :)




#459191 ĐỀ THI CHỌN HỌC SINH GIỎI TỈNH BÌNH ĐỊNH LỚP 12 THPT

Đã gửi bởi thanhdotk14 on 22-10-2013 - 11:26 trong Thi HSG cấp Tỉnh, Thành phố. Olympic 30-4. Đề thi và kiểm tra đội tuyển các cấp.

ĐỀ THI CHỌN HỌC SINH GIỎI TỈNH BÌNH ĐỊNH LỚP 12 THPT

___________________________________

 

 

Bài 1: Giải hệ phương trình:$$\left\{\begin{matrix} \frac{2xy+y\sqrt{x^2-y^2}}{14} =\sqrt{\frac{x+y}{2}}+\sqrt{\frac{x-y}{2}}& & \\ \sqrt{\left ( \frac{x+y}{2} \right )^3}+\sqrt{\left ( \frac{x-y}{2} \right )^3}=9& & \end{matrix}\right.$$

Bài 2: Cho tập $M=\left\{1,2,3,...,2013,2014\right\}$

$a.$ Lấy ngẫu nhiên ra hai số trong tập $M$. Tính xác suất để mỗi số trong hai số đó chia hết cho ít nhất một trong các số $2,3,13$

$b.$ Có bao nhiêu cách chọn ra hai tập hợp con của $M$ (không kể thứ tự) mà giao của chúng có duy nhất một phần tử ?

Bài 3: Cho dãy $(U_n)$ xác định bởi:$-1<U_0<1,U_n=\sqrt{\frac{1+U_{n-1}}{2}}$

Với $n=1,2,...$

Hai dãy $(V_n)$ và $(W_n)$ được xác định như sau:$V_n=4^n(1-U_n)$ và $W_n=U_1U_2...U_n$

Tìm $\lim V_n$ và $\lim W_n$

Bài 4: 

$1.$ Cho tam giác $ABC$. Các đường phân giác $BD,CE$ của tam giác cắt nhau tại $I$.

Chứng minh rằng: Tam giác $ABC$ vuông khi và chỉ khi $S_{BCDE}=2S_{BIC}$

$2.$ Cho hình chóp $SABC$ trong đó $SA,SB,SC$ vuông góc với nhau từng đôi một. Trên các tia $SA,SB,SC$ lần lượt lấy các điểm $A',B',C'$ sao cho: $SA.SA'=SB.SB'=SC.SC'$. Vẽ $SH\perp (A'B'C')$ cắt $(ABC)$ tại $G$

$a.$ Chứng minh rằng $G$ là trọng tâm tam giác $ABC$

$b.$ Cho $SA=a,SB=b,SC=c$. Gọi $r$ là bán kình mặt cầu nội tiếp hình chóp $SABC$.

Chứng minh:$$r=\frac{S_{SAB}+S_{SBC}+S_{SCA}-S_{ABC}}{a+b+c}$$

Bài 5: Tìm $m$ để hệ sau có nghiệm:$$\left\{\begin{matrix} \log_{x^2+y^2}(x-y)\geq 1 & & \\ x-2y=m & & \end{matrix}\right.$$




#459042 $\lim_{n\to +\infty}\sum_{k=n}^...

Đã gửi bởi thanhdotk14 on 21-10-2013 - 15:19 trong Giải tích

Tính giới hạn của tổng sau:

 

Bài 1:

 

$$L=\lim_{n\to +\infty}\sum_{k=n}^{2n}\sin\frac{\pi}{k}$$

Bài giải:

Dễ thấy:$$\sum_{k=n}^{2n} \sin \frac{\pi}{k}=\sum_{k=1}^n\sin \frac{\pi}{n+k}$$

Trước tiên ta chứng minh rằng:$$\lim_{n\to +\infty}\sum_{k=1}^n \frac{1}{n+k} =\ln 2$$

Dễ dàng chứng minh được:$$\ln (1+x)<x<-\ln (1-x), \forall 0<x<1$$

Từ đó suy ra:$\forall k=1,2,...,n$. Ta có:$$\ln \left(1+\frac{1}{n+k}\right)<\frac{1}{n+k}<-\ln \left(1-\frac{1}{n+k}\right)$$

$$\Leftrightarrow \ln (n+k+1)-\ln (n+k)<\frac{1}{n+k}<-\ln (n+k-1)+\ln (n+k)$$

Lần lượt thay $k=1,2,...,n$ và cộng vế theo vế các bất đẳng thức đó, ta được:$$\ln (2n+1)-\ln (n+1)<\sum_{k=1}^n \frac{1}{n+k}<\ln 2n-\ln n$$

$$\Leftrightarrow \ln \left(\frac{2n+1}{n+1}\right)<\sum_{k=1}^n \frac{1}{n+k} <\ln \frac{2n}{n}$$

Từ đó, theo nguyên lí kẹp, dễ dàng suy ra được:$$\lim_{n\to +\infty} \sum_{k=1}^n \frac{1}{n+k}=\ln 2(1)$$

Mặt khác, ta cũng dễ dàng chứng minh được:$$x>\sin x>x-\frac{x^3}{6} ,\forall x>0$$

Từ đó, ta suy ra được:$$\sum_{k=1}^n \frac{\pi}{n+k}>\sum_{k=1}^n \sin \frac{\pi}{n+k}>\sum_{k=1}^n \frac{\pi}{n+k}-\frac{1}{6}\sum_{k=1}^n \frac{\pi^3}{(n+k)^3}(2)$$

Dễ thấy:$$\lim_{n\to +\infty}\sum_{k=1}^{n}\frac{\pi^3}{(n+k)^3}=0(3)$$

Từ $(1),(2),(3)$ và theo nguyên lí kẹp ta suy ra được$$\lim_{n\to +\infty}\sum_{k=1}^n \sin \frac{\pi}{n+k} =\pi \ln 2$$

Hay:$$L=\pi \ln 2$$




#457283 $\frac{51}{28}\leq \frac{a}...

Đã gửi bởi thanhdotk14 on 12-10-2013 - 22:48 trong Bất đẳng thức - Cực trị



Cho a,b,c > 0 thoả mãn : $7\left ( a^{2}+b^{2}+c^{2} \right )=11(ab+bc+ca)(1)$

CMR        $\frac{51}{28}\leq \frac{a}{b+c}+\frac{b}{c+a}+\frac{c}{a+b}\leq 2$

Bài giải:

Từ $(1)$ ta suy ra được:$$7(a+b+c)^2=25(ab+bc+ca)$$

Chuẩn hóa:$a+b+c=1\Rightarrow ab+bc+ca=\frac{7}{25}$

Lại có:$$\frac{a}{b+c}+\frac{b}{c+a}+\frac{c}{a+b}$$

$$=\frac{\sum_{a,b,c}a(a+b)(a+c)}{(a+b)(b+c)(c+a)}=\frac{\frac{11}{25}+3abc}{\frac{7}{25}-abc}$$

Do đó, bất đẳng thức cần chứng minh trở thành:$$\frac{51}{28}\le \frac{\frac{11}{25}+3abc}{\frac{7}{25}-abc}\le 2$$

$$\Leftrightarrow \frac{49}{125.27}\le abc\le \frac{3}{125}$$

Mặt khác, ta thấy:$$\frac{7}{25}=a(b+c)+bc\le a(1-a)+\frac{(1-a)^2}{4}$$

$$\Rightarrow a\in \left[\frac{1}{15};\frac{3}{5}\right]$$

Tương tự, ta có:$a,b,c\in \left[\frac{1}{15};\frac{3}{5}\right]$

Do đó ta luôn có:$$\left(a-\frac{1}{15}\right) \left(b-\frac{1}{15}\right) \left(c-\frac{1}{15}\right)\ge 0$$

$$\Rightarrow abc\ge \frac{49}{125.27}$$

Đẳng thức xảy ra $\Leftrightarrow a=\frac{1}{15},b=c=\frac{7}{15}$ và các hoán vị

Tương tự ta cũng có:$$\left(a-\frac{3}{5}\right)\left(b-\frac{3}{5}\right)\left(c-\frac{3}{5}\right)\le 0$$

$$\Leftrightarrow abc\le \frac{3}{125}$$

Đẳng thức xảy ra $\Leftrightarrow a=\frac{3}{5},b=c=\frac{1}{5}$ và các hoán vị

Vậy bất đẳng thức đã được chứng minh 




#456155 Đề thi HSG thành phố Hải Phòng bảng A1 năm 2013-2014

Đã gửi bởi thanhdotk14 on 08-10-2013 - 19:35 trong Thi HSG cấp Tỉnh, Thành phố. Olympic 30-4. Đề thi và kiểm tra đội tuyển các cấp.

 

 
Bài 1. Giải hệ phương trình
$$\left\{\begin{aligned}&x^2+y^2=1\\&125y^2+\frac{6\sqrt{15}}{y^3}=125\end{aligned}\right..$$
 
 
Mathcope.o

ĐKXĐ:$y\ne 0$

Dễ dàng thấy rằng $-1\le x,y\le 1$

Nếu $y<0$ thì ta có:$\frac{6\sqrt{15}}{y^3}<0$ và $125(1-y^2)>0$

$\Rightarrow 0<y\le 1$

Khi đó ta có:$$125y^2+\frac{6\sqrt{15}}{y^3}$$

$$=\frac{125}{3}y^2+\frac{125}{3}y^2+\frac{125}{3}y^2+\frac{3\sqrt{15}}{y^3}+\frac{3\sqrt{15}}{y^3}$$

$$\ge 125 (AM-GM)$$

Khi đó, phương trình thứ hai có nghiêm khi và chỉ khi:$\frac{125}{3}y^2=\frac{3\sqrt{15}}{y^3}$

Từ đó suy ra được $y=\sqrt{\frac{3}{5}}$

 

 

 
 
 
Bài 4. Cho dãy số $\{x_n \}_{n=1}^{+\infty}$ thỏa mãn
$$x_1=20,\;x_{n+1}=\frac{1}{13}(x_n^2+7x_n+9)\quad \forall n\in\mathbb{Z}^+.$$
Đặt
$$S_n=\frac{1}{x_1+10}+\cdots+\frac{1}{x_n+10} \quad   \forall n\in\mathbb{Z}^+.$$
Chứng minh rằng $\{S_n\}_{n=1}^{+\infty}$ hội tụ và tính $\lim\limits_{n\to+\infty}S_n.$
 
 

Dễ dàng chứng minh được:$x_n\to +\infty$ khi $n\to +\infty$

Ta có:$$x_{n+1}=\frac{1}{13}(x_n^2+7x_n+9)$$

$$\Leftrightarrow 13(x_{n+1}-3)=(x_n+10)(x_n-3)$$

$$\Leftrightarrow \frac{1}{x_n+10}=\frac{1}{x_n-3}-\frac{1}{x_{n+1}-3}$$

Do đó:$$S_n=\frac{1}{x_1-3}-\frac{1}{x_{n+1}-3}$$

$$\Rightarrow \lim_{n\to +\infty} S_n=\frac{1}{17}$$

_____________

p/s:Bài 2 nhờ bạn kiểm tra lại đề nha  :icon6:  :icon6:  :icon6:




#456049 $\lim_{n\rightarrow+ \infty } \frac{x_{n+1}}{x_{1}x_...

Đã gửi bởi thanhdotk14 on 08-10-2013 - 05:59 trong Dãy số - Giới hạn

cho dãy số ($x_{n}$) được xác định như sau $x_{1}=5$ ; $x_{n+1}=x_{n}^{2}-2$     (n=1,2,....)

  tìm$\lim_{n\rightarrow+ \infty } \frac{x_{n+1}}{x_{1}x_{2}...x_{n}}$

Bài giải:

Ta có:$$x_{n+1}=x_n^2-2\Rightarrow x_n^2=x_{n+1}+2$$

$$=\frac{x_{n+1}^2-4}{x_{n+1}-2}=\frac{x_{n+1}^2-4}{x_n^2-4}$$

Do đó:$$\prod_{k=1}^n x_k^2=\frac{x_{n+1}^2-4}{x_1^2-4}=\frac{x_{n+1}^2-4}{21}$$

$$\Rightarrow \left(\frac{x_{n+1}}{x_1x_2...x_n}\right)^2=\frac{21x_{n+1}^2}{x_{n+1}^2-4}$$

Dễ dàng chứng minh được:$\lim_{n\to +\infty}x_n=+\infty$

Do đó:$$\lim_{n\to +\infty} \left(\frac{x_{n+1}}{x_1x_2...x_n}\right)^2=21$$

$$\Rightarrow \lim_{n\to +\infty} \frac{x_{n+1}}{x_1x_2...x_n}=\sqrt{21}$$




#455987 $\frac{1}{a(a+1)}+\frac{1}{...

Đã gửi bởi thanhdotk14 on 07-10-2013 - 21:53 trong Bất đẳng thức và cực trị

cho a,b,c >0 chứng minh rằng

$\frac{1}{a(a+1)}+\frac{1}{b(b+1)}+\frac{1}{c(c+1)}\geq \frac{1}{a(b+1)}+\frac{1}{b(c+1)}+\frac{1}{c(a+1)}(1)$

Bài giải:

Ta viết lại bất đẳng thức $(1)$ như sau:$$\sum_{a,b,c} \frac{b-a}{a(a+1)(b+1)}\ge 0$$

$$\Leftrightarrow \sum_{a,b,c} bc(c+1)(b-a)\ge 0$$

$$\Leftrightarrow \sum_{a,b,c} a^2b^2+\sum_{a,b,c} a^2b\ge abc(a+b+c)+3abc$$

Nhưng bất đẳng thức này luôn đúng theo $AM-GM$

Vậy bất đẳng thức được chứng minh

Đẳng thức xảy ra $\Leftrightarrow a=b=c$

__________________

@hoctronewton: chỗ màu đỏ mà bạn nói chỉ cần nhân tung tóe zô là ok à 




#455242 $\frac{a^{2}}{3}+b^{2}+c^...

Đã gửi bởi thanhdotk14 on 05-10-2013 - 06:27 trong Bất đẳng thức và cực trị

Bài này đã có ở đây




#454676 Đề thi chọn đội tuyển học sinh giỏi trường THPT chuyên Lê Quý Đôn Bình Định,...

Đã gửi bởi thanhdotk14 on 02-10-2013 - 17:37 trong Thi HSG cấp Tỉnh, Thành phố. Olympic 30-4. Đề thi và kiểm tra đội tuyển các cấp.

ĐỀ THI CHỌN ĐỘI TUYỂN HỌC SINH GIỎI LỚP 12 TRƯỜNG THPT CHUYÊN LÊ QUÝ ĐÔN 

BÌNH ĐỊNH- NĂM 2013-2014

________________________________

 

 

 

Bài 1: Cho dãy số $(U_n)$ được xác định bởi: $U_0=U_1=1, U_{n+2}=\sqrt[3]{U_{n+1}}+\sqrt[3]{U_n},\forall n\in \mathbb{N}$

Chứng minh rằng dãy $(U_n)$ có giới hạn hữu hạn. Tìm giới hạn đó.

Bài 2: Cho ba số thực $a,b,c$ biến thiên trong đoạn $[1;2]$ và thỏa mãn: $a+b+c=4$

Tìm giá trị lớn nhất của biểu thức:$$P=\frac{a}{b+c}+\frac{b}{c+a}+\frac{c}{a+b}$$

Bài 3: Có bao nhiêu bộ sắp thứ tự $(a,b,c)$, với $a,b,c$ là các số nguyên dương thỏa mãn: $[a,b,c]=2^3.3^5.5^7.7^{11}$ ? (Kí hiệu $[a,b,c]$ là bội chung nhỏ nhất của ba số $a,b,c$ nguyên dương)

Bài 4: Đường tròn nội tiếp $\Delta ABC (AB\ne AC)$, tiếp xúc với cạnh $BC,CA,AB$ tương ứng tại $D,E,F$. Đường thẳng qua $D$ và vuông góc với $FE$ cắt cạnh $AB$ tại $X$, giao điểm thứ hai của đường tròn ngoại tiếp tam giác $AEF$ và $ABC$ là $T$. Chứng minh rằng: $TX\perp TF$




#454294 $P= \frac{a^3+2}{b^2+1}+\frac{b^3+2...

Đã gửi bởi thanhdotk14 on 30-09-2013 - 20:06 trong Bất đẳng thức - Cực trị



Cho 3 số thực a,b,c thuộc $[0;1]$. Tìm GTNN của 

$P= \frac{a^3+2}{b^2+1}+\frac{b^3+2}{c^2+1}+\frac{c^3+2}{a^2+1}$

Bài giải:

Ta cần chứng minh:$$\sum_{a,b,c} \frac{a^3+2}{b^2+1}\ge \frac{9}{2}(1)$$

Ta viết lại bất đẳng thức $(1)$ như sau:$$\sum_{a,b,c} \left(\frac{a^3+2}{b^2+1}-\frac{3}{2}\right)\ge 0$$

$$\Leftrightarrow \sum_{a,b,c} \frac{2a^3-3b^2+1}{b^2+1}\ge 0(2)$$

Lại có: $2a^3+1\ge 3a^2 (AM-GM)$ nên từ $(2)$ ta cần chứng minh:$$\sum_{a,b,c} \frac{a^2-b^2}{b^2+1}\ge 0$$

$$\sum_{a,b,c} \frac{a^2+1}{b^2+1}\ge 3$$

(Vì $\frac{b^2}{b^2+1}=1-\frac{1}{b^2+1}$)

Nhưng bất đẳng thức trên luôn đúng theo $AM-GM$

Vậy bất đẳng thức được chứng minh

Đẳng thức xảy ra $\Leftrightarrow a=b=c=1$




#453754 $\frac{2a+c}{1+ac}+\frac{2b+c}...

Đã gửi bởi thanhdotk14 on 28-09-2013 - 22:24 trong Bất đẳng thức và cực trị

Cho $a,b,c$ là các số không âm thỏa $a^{2}+b^{2}+c^{2}=1$. Tìm GTLN của

$\frac{2a+c}{1+ac}+\frac{2b+c}{1+bc}+\frac{a+b+c}{1\sqrt{2}abc}$

 

Bài này, mình đã giải ở đây




#453353 Đề thi chọn đội tuyển Tp Cần Thơ 2013-2014

Đã gửi bởi thanhdotk14 on 27-09-2013 - 13:21 trong Thi HSG cấp Tỉnh, Thành phố. Olympic 30-4. Đề thi và kiểm tra đội tuyển các cấp.

Câu 2:

Ảnh chụp màn hình_2013-09-27_132317.png

$a.$ Ta có: $AE=AD.\cos \frac{A}{2}=AF$

Ta cần chứng minh:

$\frac{HB}{HC}=\frac{FB}{EC}$

$\Leftrightarrow \frac{AB.\cos B}{AC.\cos C}=\frac{BD.\cos B}{CD.\cos C}$

$\Leftrightarrow \frac{AB}{AC}=\frac{BD}{CD}$

Nhưng đẳng thức trên luôn đúng do $AD$ là đường phân giác 

Từ đó suy ra: $\frac{HB}{HC}.\frac{EC}{EA}.\frac{FA}{FB}=1$

Do đó: $AH,BE,CF$ đông quy hay ta có đpcm

$b.$ Gọi $I=FD\cap AH, J=BI\cap AC$

Ta có: $\widehat {FBJ}=\widehat{FHI}=\widehat{FDA}=\widehat{AFE}$

$\Rightarrow FE\parallel BJ$

Vì $AE=AF \Rightarrow FB=EJ$

$\Rightarrow \frac{HB}{HC}=\frac{FB}{EC}=\frac{EJ}{EC}=\frac{KB}{KC} (đpcm)$

$c.$ Ta cần chứng minh:

$FB.NB<EC.NC$

$\Rightarrow BD.\cos B.NB<CD.\cos C.NC$

$\Rightarrow (NB-ND).\cos B.NB<(NC+ND).\cos C.NC$

Vì $AB<AC\Rightarrow \cos B<\cos C$

Và $(NB-ND).NB<(NC+ND).NC$

Do đó ta có đpcm 




#452578 $x_1=1 $, $ x_n=\sqrt{x_n( x_n+1)( x_n+2)( x_n+3)+1...

Đã gửi bởi thanhdotk14 on 23-09-2013 - 18:11 trong Dãy số - Giới hạn

Bài dãy số trong đề chọn đội tuyển HSG Lâm Đồng 2013 – 2014

Cho dãy số $\left( {{x}_{n}} \right),n=1,2,3,...$ xác định bởi

$x_1=1  $

$ {{x}_{n}}=\sqrt{{{x}_{n}}\left( {{x}_{n}}+1 \right)\left( {{x}_{n}}+2 \right)\left( {{x}_{n}}+3 \right)+1}  $

$n=1,2,3,...$

a. Chứng minh : $\underset{n\to +\infty }{\mathop{\lim }}\,{{x}_{n}}=+\infty $

b. Tìm $\underset{n\to +\infty }{\mathop{\lim }}\,\sum\limits_{k=1}^{n}{\frac{1}{{{x}_{k}}+2}}$

Bài giải:

$a.$Từ cách xác định dãy, ta dễ dàng suy ra được: $x_n>0 \forall n=1,2,...$

Ta có: $$x_{n+1}=\sqrt{(x_n^2+3x_n)(x_n^2+3x_n+2)+1}$$

$$=\sqrt{(x_n^2+3x_n+1)^2}=x_n^2+3x_n+1$$

$$\Rightarrow x_{n+1}>3x_n>3^2x_{n-1}>...>3^n.x_1=3^n$$

$$\Rightarrow \lim_{n\to +\infty} x_n=+\infty$$

$b.$ Ta có:$$x_{n+1}=x_n^2+3x_n+1$$

$$\Rightarrow x_{n+1}+1=(x_n+1)(x_n+2)$$

$$\Rightarrow \frac{1}{x_{n+1}+1}=\frac{1}{x_n+1}-\frac{1}{x_n+2}$$

$$\Rightarrow \frac{1}{x_n+2}=\frac{1}{x_n+1}-\frac{1}{x_{n+1}+1}$$

Do đó: $$\sum_{k=1}^n \frac{1}{x_k+2}=1-\frac{1}{x_{n+1}+1}$$

$$\Rightarrow \lim_{n\to +\infty}\sum_{k=1}^n \frac{1}{x_k+2}=1$$

 

Bài này đã có trên tạp chí THTT năm 2005.




#452225 I,E,F thẳng hàng

Đã gửi bởi thanhdotk14 on 22-09-2013 - 08:31 trong Hình học



cho tam giác ABC nội tiếp đường tròn (O), ngoại tiếp (I) . gọi điểm D bất kỳ trên BC . Đường tròn (P) tiếp xúc với DC,DA tại E,F tiếp xúc trong với (O) tai K . Chứng minh I,E,F thẳng hàng

Bài giải:

Bổ đề 1: $AB$ là dây của đườn tròn $(O)$.$(I)$ tiếp xúc với dây $AB$ tại $K$ và tiếp xúc trong với $(O)$ tại $T$. Gọi $L$ là giao điểm của $TK$ với $(O)$. Khi đó, ta có: $L$ là trung điểm của cung $AB$ không chứa $T$ và $LA^2=LK.LT$

Ảnh chụp màn hình_2013-09-22_081128.png

Bổ đề 2: Điểm $M$ là trung điểm của cung $BC$ không chứa $A$ của đường tròn ngoại tiếp tam giác $ABC$. $I \in [MA]$ sao cho $MI=MB$.Khi đó, $I$ là tâm đường tròn nội tiếp tam giác $ABC$

Ảnh chụp màn hình_2013-09-22_081419.png

Quay lại bài toán:

Kẻ $KF$ cắt $(O)$ tại $L$, $AL$ cắt $EF$ tại $I$

Ảnh chụp màn hình_2013-09-22_080832.png

Theo bổ đề 1, ta có $AL$ là phân giác của $\widehat{BAC}$

Mặt khác, ta có: $\widehat{FEK}=\widehat{IAK}=\widehat{FKx}$

$\Rightarrow AIEK$ nội tiếp

$\Rightarrow \widehat{AIK}=\widehat{AEK}=\widehat{EFK}$

$\Delta LFI \sim \Delta LIK$

$\Rightarrow LI^2=LF.LK$

Lại theo bổ đề 1, ta có: $LC^2=LF.LK$

$\Rightarrow LI=LC$

Từ đó, theo bổ đề 2, ta suy ra: $I$ chính là tâm đường tròn nội tiếp $\Delta ABC$

Từ đó ta có đpcm

_________________________________

@kreus, zô nhận hàng nè =))




#451907 Cho a,b,c dương. CMR: $ \frac{a^2+b^2+c^2}{ab+bc+ca...

Đã gửi bởi thanhdotk14 on 20-09-2013 - 20:01 trong Bất đẳng thức và cực trị



Cho a,b,c dương. CMR: $ \frac{a^2+b^2+c^2}{ab+bc+ca} +\frac{8abc}{(a+b)(b+c)(c+a)} \geq 2(1)$

Bài giải:

Không mất tính tổng quát ta có thể giả sử $c=\min\left\{a;b;c\right\}$

Ta viết lại bất đẳng thức $(1)$ như sau:$$\frac{a^2+b^2+c^2}{ab+bc+ca}-1+\frac{8abc}{(a+b)(b+c)(c+a)}-1\ge 0$$

$$\Leftrightarrow \prod_{a,b,c} (a+b). [(a-b)^2+(a-c)(b-c)]-(ab+bc+ca)[2c(a-b)^2+(a+b)(a-c)(b-c)]\ge 0$$

$$\Leftrightarrow M(a-b)^2+N(a-c)(b-c)\ge 0$$

Với $M=\prod_{a,b,c} (a+b)-2c(ab+bc+ca), N=\prod_{a,b,c} (a+b)-(a+b)(ab+bc+ca)$

Đến đây ta dễ dàng chứng minh được rằng $M,N\ge 0$ với $c=\min\left\{a;b;c\right\}$

Do đó, bất đẳng thức được chứng minh

Đẳng thức xảy ra $\Leftrightarrow a=b=c$




#451668 $ (a+b-c-1)(b+c-a-1)(c+a-b-1) \le 8 $

Đã gửi bởi thanhdotk14 on 19-09-2013 - 15:44 trong Bất đẳng thức - Cực trị



Bài 12: $x,y,z$ là 3 số thực phân biệt. CMR:

$\sum \left ( \frac{a-b}{b-c} \right )^2>\sum \frac{a+b}{b+c}(1)$

Bài giải:

Đặt $(a+b;b+c;c+a)\to (x;y;z)$ ($x\ne y\ne z\ne 0$)

$\Rightarrow \left\{\begin{matrix} a=\frac{x-y+z}{2} & & \\ b=\frac{x+y-z}{2} & & \\ c=\frac{-x+y+z}{2} & & \end{matrix}\right.$

Từ đó, $(1)$ được viết lại thành: $$\sum \left(\frac{z-y}{x-z}\right)^2\ge \sum \frac{x}{y}(2)$$

Lại có: $$\left(\frac{z-y}{x-z}\right)^2-\frac{x}{y}=\frac{(y-x)(y-z)(y-z+x)}{y(x-z)^2}-\frac{x(x-y)(x-z)}{y(x-z)^2}$$

Do đó: $(2)$ được viết lại thành:$$\sum \frac{(y-x)(y-z)(y-z+x)}{y(x-z)^2}-\sum \frac{x(x-y)(x-z)}{y(x-z)^2}>0$$

$$\Leftrightarrow \sum (x-y)(x-z)\left[\frac{x-y+z}{x(z-y)^2}-\frac{x}{y(x-z)^2}\right]>0$$

$$\Leftrightarrow A(x-y)(x-z)+B(y-x)(y-z)+C(z-x)(z-y)>0$$

Với $A=\frac{x-y+z}{x(z-y)^2}-\frac{x}{y(x-z)^2}, B=..., C=...$  :icon6:  :icon6:  :icon6:

Giả sử $(x;y;z)$ đơn điệu thì $(A;B;C)$ cũng đơn điệu (Tự suy ra nha, lười gõ quá)  >:)  >:)  >:)

Từ đó áp dụng bất đẳng thức schur suy rộng ta có ngay đpcm




#451113 $x_0=a, x_{n+1}=2-x_n^2$

Đã gửi bởi thanhdotk14 on 17-09-2013 - 04:51 trong Dãy số - Giới hạn

Bài toán: Tìm tất cả các giá trị của $a$ để dãy số $\left\{x_n\right\}$ xác định bởi $x_0=a, x_{n+1}=2-x_n^2$ có giới hạn hữu hạn.